Fran is training for her first marathon, and she wants to know if there is a significant difference between the mean number of miles run each week by group runners and individual runners who are training for marathons. She interviews 42 randomly selected people who train in groups, and finds that they run a mean of 47.1 miles per week. Assume that the population standard deviation for group runners is known to be 4.4 miles per week. She also interviews a random sample of 47 people who train on their own and finds that they run a mean of 48.5 miles per week. Assume that the population standard deviation for people who run by themselves is 1.8 miles per week. Test the claim at the 0.01 level of significance. Let group runners training for marathons be Population 1 and let individual runners training for marathons be Population 2. Draw a conclusion and interpret the decision.
A. We fail to reject the null hypothesis and conclude that there is insufficient evidence at a 0.10 level of significance to support Fran's claim that there is a significant difference between the mean number of miles run each week by group runners and individual runners.
B. We reject the null hypothesis and conclude that there is insufficient evidence at a 0.10 level of significance to support Fran's claim that there is a significant difference between the mean number of miles run each week by group runners and individual runners.
C.We fail to reject the null hypothesis and conclude that there is sufficient evidence at a 0.10 level of significance to support fran's claim that there is a significant difference between the mean number of miles run each week by group runners and individual runners.
D. We reject the null hypothesis and conclude that there is sufficient evidence at a 0.10 level of significance to support Fran's claim that there is a significant difference between the mean number of miles run each week by group runners and individual runners.

Answers

Answer 1

Answer:Sorry i have no time to read it all bro

Step-by-step explanation:


Related Questions

PLEASE HELP ASAP
Mr. Green has a class of 15 students. He can spend $20 on each student to buy math supplies for the year. He first buys all of his students calculators, which
costs a total of $144.75. After buying the calculators, how much does he have left to spend on each student?

Answers

Answer:

10.35

Step-by-step explanation:

He has $300 (15x$20). He spends 144.75. He has 155.25 in total. That leaves 10.35 (155.25/15) to spend on each student.

A property has been assessed at $225 000. The mill rate is 14.5. To find the property tax, you would multiply the mill rate by: *
a) 0.10
b) 0.001
c) 0.01
d) 0.0001

Answers

A would be the answer

To find the property tax, you would multiply the mill rate by 0.01

To find the property tax, you would multiply the assessed value of the property by the mill rate.

Given that the assessed value is $225,000 and the mill rate is 14.5, we need to multiply these two values to calculate the property tax.

Property Tax = Assessed Value × Mill Rate

Property Tax = $225,000 × 14.5

To simplify the calculation, we can convert the mill rate to a decimal by dividing it by 1,000.

Mill Rate = 14.5 ÷ 1000 = 0.0145

Property Tax = $225,000 × 0.0145

Property Tax = $3,262.50

Therefore, to find the property tax, you would multiply the mill rate by 0.01 (option c).

To know more about property tax click here :

https://brainly.com/question/17168258

#SPJ2

PLS HELP FOR 40 POINTS I HAVE AN EXAM!!!

You need to select a pair of socks from the draw and then a t-shirt. You have eight different colors of sock pairs and ten different color t-shirts. If you pull one from each draw, what are the possible combinations of socks and t-shirt that you will get?

Answers

Answer:

8

Step-by-step explanation:

You only have 8 socks and its a combination of both socks and t-shirts.  Hope this helps!

Answer:

80

Step-by-step explanation:

10*8=80

Help please :) love yall. select three choices that are ways to show 0.48. last one is forty-eight tenths ​

Answers

Answer: The first, The second, and the number line one love!

Step-by-step explanation:

30 POINTS AND BRAINLIEST Alexis needs to paint the four exterior walls of a large rectangular barn. The length of the barn is 80 feet, the width is 50 feet, and the height is 30 feet. The paint costs $28 per gallon, and each gallon covers 420 square feet. What is the area of the barn Alexis needs to paint? ______________ Explain how you found your answer.

Answers

Answer:

It will cost $532 to paint the barn

Step-by-step explanation:

First we need to find the area of the barn's walls.

2 (length x height) + 2 (width x height)

=2 (80x30) + 2 (50x30)

=2 (2400) + 2(1500)

=4800+3000

=7800 ft^2

Now we can find the amount of gallons needed, then the total cost.

7800 ft^2/420 ft^2  

=18.57 gallons

=19 gallons  

19 gallons x $28

=$532

Answer:

520

Step-by-step explanation:

Area:

[tex]2(80*30) + 2(50*30) = 7800[/tex]

Price

[tex]\frac{7800}{420}*28 = 520[/tex]

Let u=ln(x) and v=ln(y). Write the expression below in terms of u and v

Answers

Answer:  (1/2)u - v

=======================================================

Work Shown:

We'll apply these log rules

[tex]\text{Log Rule 1: } \ \ \ \ln\left(\frac{x}{y}\right) = \ln(x) - \ln(y)\\\\[/tex]

[tex]\text{Log Rule 2: } \ \ \ \ln\left(x^y\right) = y\ln(x)\\\\[/tex]

These log rules can be used for logs of any base, and not just natural logs.

[tex]z = \ln\left(\frac{\sqrt{x}}{y}\right)\\\\z = \ln\left(\sqrt{x}\right)-\ln\left(y\right) \text{ ... Use log rule 1}\\\\z = \ln\left(x^{1/2}\right)-\ln\left(y\right)\\\\z = \frac{1}{2}\ln\left(x\right)-\ln\left(y\right) \text{ ... Use log rule 2}\\\\z = \frac{1}{2}u-v\\\\[/tex]

Therefore,

[tex]\ln\left(\frac{\sqrt{x}}{y}\right) = \frac{1}{2}u-v\\\\[/tex]

when [tex]u = \ln(x) \ \text{ and } \ v = \ln(y)\\\\[/tex]

Jaylin Earned $36 for 3 hours of work and $60 for 5 hours of work. What is the Constant of Proportionality?

Answers

I believe it would be 12$ per hour or aka constant proportionality!! I hope I get brain-list!!
Explanation
36 divide by 3 = 12
60 divide by 5 = 12
So twelve is your answer

. A point P is equidistant from R(-2, 4) and S(6,-4)
and its x-coordinate is twice its y-coordinate.
(0) Find the coordinates of P.
(ii) Hence, determine whether P, R and S are
collinear, showing your working clearly.​

Answers

Answer:

I) coordinate of P is (11, 5.5)

II) Points are not collinear

Step-by-step explanation:

I) Let the coordinates of P be (a, b)

We are told that its x-coordinate is twice its y-coordinate. Thus, a = 2b

Now, P is equidistant from R(-2, 4) and S(6,-4). Thus;

PR = PS

Using the formula for distance between two coordinates, we have;

√((a - (-2))² + (b - 4)²) = √((a - 6)² + (b - (-4))²)

This gives;

(a + 2)² + (b - 4)² = (a - 6)² + (b + 4)²

Since a = 2b,then we have;

(2b + 2)² + (b - 4)² = (2b - 6)² + (b + 4)²

4b² + 8b + 4 + b² - 8b + 16 = 4b² - 12b + 36 + b² + 8b + 16

Simplifying this gives;

20 = 42 - 4b

42 - 20 = 4b

4b = 22

b = 22/4

b = 11/2

b = 5.5

Since a = 2b

Then a = 2 × 5.5

a = 11

Thus, coordinate of P is (11, 5.5)

II) Formula for area of triangle formed by three points is given as;

A = ½|(x1 - x2) (x2 - x3)|

|(y1 - y2) (y2 - y3)|

x1 - x2 = 11 - (-2) = 13

x2 - x3 = -2 - 6 = -8

y1 - y2 = 5.5 - 4 = 1.5

y2 - y3 = 4 - (-4) = 8

Thus;

A = ½|13 -8|

|1.5 8|

A = ½((13 × 8) - (1.5 × - 8))

A = ½(104 + 12)

A = 58

Since the area is not zero, then it means the points are not collinear.

I Need helppp! Please helpp!!

Answers

9514 1404 393

Answer:

  x = 1/2

Step-by-step explanation:

There are some different ways to find the area of a donut shape. One is to multiply its centerline length by its width. We'll use that here to find the shaded area of the sector.

The centerline radius of shaded area A is ...

  (6x -2) cm + (1/2)(4x cm) = 8x -2 cm

The length of the centerline is ...

  s = rθ = (8x -2)(π/4) . . . . . . for a central angle of 45° = π/4 radians

Then the area A is ...

  area A = (π/4)(8x -2)(4x) = 2πx(4x -1) . . . cm^2

__

The radius of circle B is ((4 -4x) cm)/2 = (2 -2x) cm. The area of circle B is ...

  A = πr^2 = π(2 -2x)^2 cm^2

We're told the two areas are equal, so we have ...

  2πx(4x -1) = π(2 -2x)^2 . . . . . . . expressions for the areas are equal

  8x^2 -2x = 4x^2 -8x +4 . . . . . . divide by π, eliminate parentheses

  4x^2 +6x -4 = 0 . . . . . . . . . . . . subtract the right-side expression

  2(2x -1)(x +2) = 0 . . . . . . . . . . . factor

  x = 1/2 or -2 . . . . . . . only the positive solution is useful here

The value of x is 1/2.

HELP ME PLEASEEEEEEEEEEEE

Answers

Answer: 20 cm²

Step-by-step explanation:

A billing company that collects bills for​ doctors' offices in the area is concerned that the percentage of bills being paid by medical insurance has risen.​ Historically, that percentage has been 31​%. An examination of 8,606 ​recent, randomly selected bills reveals that 32​% of these bills are being paid by medical insurance. Is there evidence of a change in the percent of bills being paid by medical​ insurance

Answers

Answer:

Following are the responses to the given question:

Step-by-step explanation:

[tex]H_o : p =0.31\\\\H_a: p \neq 0.31\\\\n = 8606 \\\\ p^ = 0.32[/tex]

[tex]TS = \frac{(p^ - p)}{\sqrt{(\frac{pq}{n})}}[/tex]

      [tex]= \frac{(0.32-0.31)}{\sqrt{(\frac{0.31\times 0.69}{8368})}}\\\\=\frac{0.01}{5.4}\\\\=0.00185\\\\[/tex]

Each quarter the marketing manager of a retail store divides customers into two classes based on their purchase behavior in the previous quarter. Denote the classes as L for low and H for high. The manager wishes to determine to which classes of customers he should send quarterly catalogs. The cost of sending a catalog is $15 per customer and the expected purchase depends on the customer’s class and the manager’s action. If a customer is in class L and receives a catalog, then the expected purchase in the current quarter is $20, and if a class L customer does not receive a catalog his expected purchase is $10. If a customer is in class H and receives a catalog, then his expected purchase is $50, and if a class H customer does not receive a catalog his expected purchase is $25. The decision whether or not to send a catalog to a customer also affects the customer's classification in the subsequent quarter. If a customer is class L at the start of the present quarter, then the probability he is in class L at the subsequent quarter is 0.3 if he receives a catalog and 0.5 if he does not. If a customer is class H in the current period, then the probability that he remains in class H in the subsequent period is 0.8 if he receives a catalog and 0.4 if he does not. Assume a discount rate of 0.9 and an objective of maximizing expected total discounted reward.
a) Formulate this as an infinite-horizon discounted Markov decision problem.
b) For ???? = 0.1, find a near-optimal policy using value iteration.
c) Find an optimal policy using policy iteration starting with the stationary policy which has greatest one-step reward.
d) Formulate the problem as a linear program, giving its primal and dual. Solve both and interpret the solutions.

Answers

Answer:

55

Step-by-step explanation:

The height of the equilateral triangle below is 15 units. What is the value of x?

An equilateral triangle is shown. A perpendicular bisector cuts the triangle into 2 equal parts. They hypotenuse has a measure of x.
15 StartRoot 3 EndRoot
30 StartRoot 3 EndRoot
StartFraction 15 Over StartRoot 3 EndRoot EndFraction
StartFraction 30 Over StartRoot 3 EndRoot

Answers

Answer: b

Step-by-step explanation: on edge

Answer: C. 30 StartRoot 3 EndRoot

Step-by-step explanation:

Took the test/exam

Help me please can someone tell me how to do this

Answers

Independent, should be the answer.

URGENT NEED HELP CLICK TO SEE

Answers

Answer:

4.20

Step-by-step explanation:

First find the total cost of the nametags

105 * .12

12.60

Then divide by the 3 teachers

12.60/3 =4.20

Each teacher will pay 4.20

Help please and explain how to get the answerm

Answers

Answer:

147 degrees

Step-by-step explanation:

all the angles of a triangle add up to 180 degrees. so, you must perform the operation 180-(83+64) which equals 33. this is the angle supplementary to angle 1. to find angle 1, you must subtract 180 and 33 since supplementary angles add up to 180 degrees. this is equal to 147 degrees.

Given the piecewise function below, which equation represents piece "B"?

Answers

Answer:

y=1

Step-by-step explanation:

x is 0 so there is no point putting that in in the equation y=mx+b you do have the b which is 1 so that is your equation

Write one way of representing the equation of the given line in point-slope form. Then rewrite the equation in slope-intercept form.

Answers

answer:

step-by-step explanation:
points (0, 3) and (7, 4)
point-slope form = y - y1 = m(x - x1)
y - 3 = 1(x - 0)
just add y and x from either point to complete the form
slope-intercept form = y = mx + b
y = (0, -3)
m = 1
y = x - 3
x is the slope and -3 is the y-intercept

Is this is a math question?

Answers

yes it is
it says mathematics 5 points

Which factored form of 10x + 120x + 320 reveals all the zeros of the function it defines?
A) 10(x + 6)^2 – 40
B) 10(x + 8) (x + 4)
C) 10x (x + 12) + 320
D) 10(x^2 + 12x +32)

Answers

Answer:

b

Step-by-step explanation:

Solve for x 5x - 10 = -10​

Answers

Answer:

the answer is x = 0

Step-by-step explanation:

Cancel equal terms on both sides of the equation:

[tex]5x = 0[/tex]

Divide both side of the equation by 5.

[tex]x = 0[/tex]

Therefore, the answer is x = 0

Answer:

5x - 10 = -10

+10 +10

5x = 0

divide by 5 on both sides:

x = 0

Ms jaegar can type 280 words in 4 minutes on her computer. If this is her average rate, How much word will she type in 10 minutes?
Show your work

Answers

Answer:

700 words

Step-by-step explanation:

280: 4

x:10

280/4 = x/10

4times x is equal to 4x and 280 times 10 is equal to 2800

4x=2800

2800 divided by x is 700.

The answer is 700 words per 10 minutes.


280x2=560. 8min

280/2= 140. 2min.

560+140=700. 10min.

Can someone help me out

Answers

Answer= i believe it’s gonna be “100” as well

Sarah's dad bought her a scooter that cost $89.95. He figures he can spend up to $30.05 on a helmet before he spends too much money in all. Let x represent how much money Sarah's dad wants to spend in all. Which inequality describes the problem?

Answers

Answer:

x - 89.95 ≤ 30.05= x ≤ 120

add 89.95 to both sides of the equation. Since on the right side 89.95 is already there it cancells eachother out leaving x. After adding 89.95 to 30.05 you should get an answer of 120.

The money Sarah's dad wants to spend in all if Sarah's dad bought her a scooter that costs $89.95, He can spend up to $30.05 on a helmet, which is less than or equal to $120, x ≤ 120.

What is inequality?

Inequality is a relationship that compares two numbers or other mathematical expressions in an unequal way. Inequalities are the name given to certain algebraic mathematical expressions.

Given:

Sarah's dad bought her a scooter that cost $89.95,

He can spend up to $30.05 on a helmet,

Assume the x is the money Sarah's dad wants to spend in all,

Write the inequality as shown below,

x - 89.95 ≤ 30.05

Solve the inequality by adding 89.95 on both sides,

x - 89.95  + 89.95   ≤ 30.05 + 89.95  

x ≤ 120

To know more about inequality:

https://brainly.com/question/2038369

#SPJ2

Solve triangle ABC
B=56 degrees, a=14

Answers

I believe B - 56 and the other two would be 17? but I’m not completely sure

Write the explicit equation.
-8, -2, 4, 10

Answers

Answer:

a*n = -8 + (n - 1)6

Step-by-step explanation:

Explicit formula:

a*n = a*1 + (n - 1)d

a*1 = first term

n = nth term or the place

d = difference

Set: -8, -2, 4, 10

-8 ---> -2 = 6 added

Fill it in.

a*1 = -8

n = n

d = 6

Solution:

a^n = -8 + (n -1)6

4) Jenee made a 99, 86, 76, and 95 on her first four spelling tests. Her mother told her
that her spelling test average had to be a 91 for Jenee to get a new video game. What
does Jenee need to make on her next test?

Answers

Answer: She needs to get 99 on her next test.

Step-by-step explanation:

Find the 8th term in the
sequence
-1/2,-1, -2, -4,...
Hint: Write a formula to help you.
1st term . Common Ratio desired term – 1)

Answers

Answer:

-64

Step-by-step explanation:

By looking at the numbers, we can identify that this is a geometric sequence and the ratio would be 2. The formula to find the nth term in a geometric series is:

[tex]t *r^{n-1}[/tex]

where t is the first term and r is the ratio. We can substitute in the information we know:

[tex]-\frac{1}{2} *2^{n-1}[/tex]

We are looking for the eighth term, so we substitute 8 where n is:

[tex]-\frac{1}{2}*2^{8-1}[/tex]

We can simplify:

[tex]-\frac{1}{2}*2^7[/tex]

[tex]-2^6[/tex]

[tex]-64[/tex]

If F(x) = +4, which of the following is the inverse of Fx)?
O A. F^1(x) = 2(x-4)
O B. F^1(x) = 7(x+4)
O C.F^1(x) = 7(x-4)
D. F1(x) = 2(x+4)​

Answers

Answer:

[tex]f^{-1} = \frac{ 7(x-4)}{2}[/tex]

Step-by-step explanation:

Given

[tex]f(x) = \frac{2x}{7} + 4[/tex]

Required

The inverse function

[tex]f(x) = \frac{2x}{7} + 4[/tex]

Replace f(x) with y

[tex]y = \frac{2x}{7} + 4[/tex]

Swap x and y

[tex]x = \frac{2y}{7} + 4[/tex]

Multiply through by 7

[tex]7x = 2y + 28[/tex]

Solve for 2y

[tex]2y = 7x - 28[/tex]

Solve for y

[tex]y = \frac{ 7x-28}{2}[/tex]

Factorize

[tex]y = \frac{ 7(x-4)}{2}[/tex]

Write as:

[tex]f^{-1} = \frac{ 7(x-4)}{2}[/tex]

help me please. thank you if you do​

Answers

Answer:

B

Step-by-step explanation:

Other Questions
The square pyramid below represents a tent that Wayne is making. The sides of the tent will be covered in blue fabric and the base of the tent will be covered in brown fabric. Find the surface area that will be covered in each color of fabric. Im not good with math okay?? A runner ran 2/3 of a race in 21 minutes. They ran the entire race at a constant speed How long did it take to run the entire race? If I have a 64 in English, what would I need to get on my final exam to pass with an 80? Parker borrowed some money from his friend in order to help buy a new video game system. Parker agreed to pay the friend back some amount every week until his loan was paid off. An equation representing the amount Parker owes his friend after tt weeks is L=150-25t. What is the y-intercept of the equation and what is its interpretation in the context of the problem? 12. The least-squares regression equation = 10 + 2x describes the relationship between x = age (inweeks) and y = weight (in pounds) for a sample of 50 German shepherd puppies for ages 5 weeks. The weight of a particular German Shepherd puppy that is 10 weeks old is underestimated by 10pounds. How much does this particular puppy weigh?(A) 10 pounds(B)20 pounds(C) 30 pounds(D) 40 pounds(E) 50 pounds Whats the correct answer????? Correct answer gets brainlest I need an essay on UN sustainable goal 2 zero hunger ASAP.PLEASE HELP A jeweler had 38 inches of silver chain. She need five times that much to make some necklaces and 3 times that amount to make some bracelets. How much silver chain does the jeweler need to make her necklaces and bracelets? need or want Help please GIVING BRAINLY AND THIS IS A BASKETBALL QUESTIONList and describe each these basketball terms:Jump BallAssistStealLay UpTechnical Foul Find the sum of the polynomials -4x^2+9x-3 and 7x^2-5x+4 1) Color blindness is a trait that is carried on the X chromosome. Perform a cross between the following parents: a) A woman who has normal vision but whose father was colorblind. b) A man who is colorblind2) If these individuals have exactly four children together, what number of children would you expect to be male, female, colorblind, and not colorblind? Translate: to celebratefestejarO bautizarO comulgarcumplir aos c Write a program that simulates a magic square using 3 one dimensional parallel arrays of integer type. Each one the arrays corresponds to a row of the magic square. The program asks the user to enter the values of the magic square row by row and informs the user if the grid is a magic square or not. flowchart Can someone tell me the actual way to say "where shall we meet?" please 45 points Multiple Choice: Choose the answer that best fits each statement below.______ 5. Which of the following can be found by clicking the AutoSum dropdown?a. Averageb. Minc. Sumd. All of the above______ 6. Which option is used to prevent a cell reference from changing when a formula is copied toanother location?a. Named rangesb. Absolute cell reference______ 7. An advantage to defining range names is:a. Selections can be largerb. Selections can be any formatc. Name ranges are easy to rememberd. Name ranges clear cell contentsTrue/False: Answer True or False for each statement below.______ 8. You can only increase or decrease the decimal places by two.______ 9. The comma style allows you to format with a thousands separator.______ 10. Excel does not allow you to copy and paste formulas with AutoFill. A water tank is in the shape of a right circular cylinder with a height of 20 feet a volume of 320 cubic feet. What is the diameter, in feet, of the water tank?A, 16B, 10C, 8D, 4 Question 3 of 11 Lucy asked 5 friends how many books they each read last month. She put her data in a table and found the total books read. Friend Ben Ann Dan Ellie Caty Total Number of books 0 0 3 3 10 Find the mean and the median of the data set. Determine which of these values is greater. A. The median, 3, is greater than the mean, 2. B. The median, 4, is greater than the mean, 0. . C. The mean, 3. is greater than the median, 2 D. The mean, 4, is greater than the median, 3. 3. Mark the figurative language technique used: Sometimes it lasts in love / 1 pointBut sometimes it hurts instead" ('Someone Like You'. Adele. 2011)a) Metaphorb) Similec) Alliterationd) Juxtaposition